What does it mean for the cross product to be invariant under orthogonal transformation?












0












$begingroup$


This article claims $Ax times Ay = det(A)(xtimes y)$, but using the vectors $(3,2,1)$ and $(5,4,6)$ and the orthogonal transformation with determinant $1$:



$$left(begin{matrix}
frac{1}{2} & frac{-sqrt{3}}{2} & 0 \
frac{sqrt{3}}{2} & frac{1}{2} & 0 \
0 & 0 & 1
end{matrix}right)$$

does not hold.



$$left(begin{matrix}
frac{1}{2} & frac{-sqrt{3}}{2} & 0 \
frac{sqrt{3}}{2} & frac{1}{2} & 0 \
0 & 0 & 1
end{matrix}right)left(begin{matrix}
3 \
2 \
1
end{matrix}right)=left(begin{matrix}
frac{-2*sqrt{3}+3}{2} \
frac{3*sqrt{3}+2}{2} \
1
end{matrix}right)$$



$$left(begin{matrix}
frac{1}{2} & frac{-sqrt{3}}{2} & 0 \
frac{sqrt{3}}{2} & frac{1}{2} & 0 \
0 & 0 & 1
end{matrix}right)left(begin{matrix}
5 \
4 \
6
end{matrix}right)=left(begin{matrix}
frac{-4*sqrt{3}+5}{2} \
frac{5*sqrt{3}+4}{2} \
6
end{matrix}right)$$



and
$$left(begin{matrix}
frac{-2*sqrt{3}+3}{2} \
frac{3*sqrt{3}+2}{2} \
1
end{matrix}right)times left(begin{matrix}
frac{-4*sqrt{3}+5}{2} \
frac{5*sqrt{3}+4}{2} \
6
end{matrix}right)=left(begin{matrix}
frac{13*sqrt{3}+8}{2} \
frac{8*sqrt{3}-13}{2} \
2
end{matrix}right)$$



but $(3,2,1)times (5,4,6)=(8,-13,2)$.



So, what does it mean for the cross product to be invariant under orthogonal transformation?










share|cite|improve this question











$endgroup$








  • 1




    $begingroup$
    The claim in the linked note is obviously wrong. I think it is correct if you multiply the cross-product on the right by A.
    $endgroup$
    – Lukas Geyer
    Jan 28 at 19:53






  • 1




    $begingroup$
    You say "this article" : first of all, it is not an article, it is a page page written by anybody anywhere in the Realm of nowhere... It is not because something is written that it is true...
    $endgroup$
    – Jean Marie
    Jan 28 at 20:41


















0












$begingroup$


This article claims $Ax times Ay = det(A)(xtimes y)$, but using the vectors $(3,2,1)$ and $(5,4,6)$ and the orthogonal transformation with determinant $1$:



$$left(begin{matrix}
frac{1}{2} & frac{-sqrt{3}}{2} & 0 \
frac{sqrt{3}}{2} & frac{1}{2} & 0 \
0 & 0 & 1
end{matrix}right)$$

does not hold.



$$left(begin{matrix}
frac{1}{2} & frac{-sqrt{3}}{2} & 0 \
frac{sqrt{3}}{2} & frac{1}{2} & 0 \
0 & 0 & 1
end{matrix}right)left(begin{matrix}
3 \
2 \
1
end{matrix}right)=left(begin{matrix}
frac{-2*sqrt{3}+3}{2} \
frac{3*sqrt{3}+2}{2} \
1
end{matrix}right)$$



$$left(begin{matrix}
frac{1}{2} & frac{-sqrt{3}}{2} & 0 \
frac{sqrt{3}}{2} & frac{1}{2} & 0 \
0 & 0 & 1
end{matrix}right)left(begin{matrix}
5 \
4 \
6
end{matrix}right)=left(begin{matrix}
frac{-4*sqrt{3}+5}{2} \
frac{5*sqrt{3}+4}{2} \
6
end{matrix}right)$$



and
$$left(begin{matrix}
frac{-2*sqrt{3}+3}{2} \
frac{3*sqrt{3}+2}{2} \
1
end{matrix}right)times left(begin{matrix}
frac{-4*sqrt{3}+5}{2} \
frac{5*sqrt{3}+4}{2} \
6
end{matrix}right)=left(begin{matrix}
frac{13*sqrt{3}+8}{2} \
frac{8*sqrt{3}-13}{2} \
2
end{matrix}right)$$



but $(3,2,1)times (5,4,6)=(8,-13,2)$.



So, what does it mean for the cross product to be invariant under orthogonal transformation?










share|cite|improve this question











$endgroup$








  • 1




    $begingroup$
    The claim in the linked note is obviously wrong. I think it is correct if you multiply the cross-product on the right by A.
    $endgroup$
    – Lukas Geyer
    Jan 28 at 19:53






  • 1




    $begingroup$
    You say "this article" : first of all, it is not an article, it is a page page written by anybody anywhere in the Realm of nowhere... It is not because something is written that it is true...
    $endgroup$
    – Jean Marie
    Jan 28 at 20:41
















0












0








0





$begingroup$


This article claims $Ax times Ay = det(A)(xtimes y)$, but using the vectors $(3,2,1)$ and $(5,4,6)$ and the orthogonal transformation with determinant $1$:



$$left(begin{matrix}
frac{1}{2} & frac{-sqrt{3}}{2} & 0 \
frac{sqrt{3}}{2} & frac{1}{2} & 0 \
0 & 0 & 1
end{matrix}right)$$

does not hold.



$$left(begin{matrix}
frac{1}{2} & frac{-sqrt{3}}{2} & 0 \
frac{sqrt{3}}{2} & frac{1}{2} & 0 \
0 & 0 & 1
end{matrix}right)left(begin{matrix}
3 \
2 \
1
end{matrix}right)=left(begin{matrix}
frac{-2*sqrt{3}+3}{2} \
frac{3*sqrt{3}+2}{2} \
1
end{matrix}right)$$



$$left(begin{matrix}
frac{1}{2} & frac{-sqrt{3}}{2} & 0 \
frac{sqrt{3}}{2} & frac{1}{2} & 0 \
0 & 0 & 1
end{matrix}right)left(begin{matrix}
5 \
4 \
6
end{matrix}right)=left(begin{matrix}
frac{-4*sqrt{3}+5}{2} \
frac{5*sqrt{3}+4}{2} \
6
end{matrix}right)$$



and
$$left(begin{matrix}
frac{-2*sqrt{3}+3}{2} \
frac{3*sqrt{3}+2}{2} \
1
end{matrix}right)times left(begin{matrix}
frac{-4*sqrt{3}+5}{2} \
frac{5*sqrt{3}+4}{2} \
6
end{matrix}right)=left(begin{matrix}
frac{13*sqrt{3}+8}{2} \
frac{8*sqrt{3}-13}{2} \
2
end{matrix}right)$$



but $(3,2,1)times (5,4,6)=(8,-13,2)$.



So, what does it mean for the cross product to be invariant under orthogonal transformation?










share|cite|improve this question











$endgroup$




This article claims $Ax times Ay = det(A)(xtimes y)$, but using the vectors $(3,2,1)$ and $(5,4,6)$ and the orthogonal transformation with determinant $1$:



$$left(begin{matrix}
frac{1}{2} & frac{-sqrt{3}}{2} & 0 \
frac{sqrt{3}}{2} & frac{1}{2} & 0 \
0 & 0 & 1
end{matrix}right)$$

does not hold.



$$left(begin{matrix}
frac{1}{2} & frac{-sqrt{3}}{2} & 0 \
frac{sqrt{3}}{2} & frac{1}{2} & 0 \
0 & 0 & 1
end{matrix}right)left(begin{matrix}
3 \
2 \
1
end{matrix}right)=left(begin{matrix}
frac{-2*sqrt{3}+3}{2} \
frac{3*sqrt{3}+2}{2} \
1
end{matrix}right)$$



$$left(begin{matrix}
frac{1}{2} & frac{-sqrt{3}}{2} & 0 \
frac{sqrt{3}}{2} & frac{1}{2} & 0 \
0 & 0 & 1
end{matrix}right)left(begin{matrix}
5 \
4 \
6
end{matrix}right)=left(begin{matrix}
frac{-4*sqrt{3}+5}{2} \
frac{5*sqrt{3}+4}{2} \
6
end{matrix}right)$$



and
$$left(begin{matrix}
frac{-2*sqrt{3}+3}{2} \
frac{3*sqrt{3}+2}{2} \
1
end{matrix}right)times left(begin{matrix}
frac{-4*sqrt{3}+5}{2} \
frac{5*sqrt{3}+4}{2} \
6
end{matrix}right)=left(begin{matrix}
frac{13*sqrt{3}+8}{2} \
frac{8*sqrt{3}-13}{2} \
2
end{matrix}right)$$



but $(3,2,1)times (5,4,6)=(8,-13,2)$.



So, what does it mean for the cross product to be invariant under orthogonal transformation?







linear-algebra cross-product






share|cite|improve this question















share|cite|improve this question













share|cite|improve this question




share|cite|improve this question








edited Jan 28 at 19:58









José Carlos Santos

171k23132240




171k23132240










asked Jan 28 at 19:45









Al JebrAl Jebr

4,37943378




4,37943378








  • 1




    $begingroup$
    The claim in the linked note is obviously wrong. I think it is correct if you multiply the cross-product on the right by A.
    $endgroup$
    – Lukas Geyer
    Jan 28 at 19:53






  • 1




    $begingroup$
    You say "this article" : first of all, it is not an article, it is a page page written by anybody anywhere in the Realm of nowhere... It is not because something is written that it is true...
    $endgroup$
    – Jean Marie
    Jan 28 at 20:41
















  • 1




    $begingroup$
    The claim in the linked note is obviously wrong. I think it is correct if you multiply the cross-product on the right by A.
    $endgroup$
    – Lukas Geyer
    Jan 28 at 19:53






  • 1




    $begingroup$
    You say "this article" : first of all, it is not an article, it is a page page written by anybody anywhere in the Realm of nowhere... It is not because something is written that it is true...
    $endgroup$
    – Jean Marie
    Jan 28 at 20:41










1




1




$begingroup$
The claim in the linked note is obviously wrong. I think it is correct if you multiply the cross-product on the right by A.
$endgroup$
– Lukas Geyer
Jan 28 at 19:53




$begingroup$
The claim in the linked note is obviously wrong. I think it is correct if you multiply the cross-product on the right by A.
$endgroup$
– Lukas Geyer
Jan 28 at 19:53




1




1




$begingroup$
You say "this article" : first of all, it is not an article, it is a page page written by anybody anywhere in the Realm of nowhere... It is not because something is written that it is true...
$endgroup$
– Jean Marie
Jan 28 at 20:41






$begingroup$
You say "this article" : first of all, it is not an article, it is a page page written by anybody anywhere in the Realm of nowhere... It is not because something is written that it is true...
$endgroup$
– Jean Marie
Jan 28 at 20:41












1 Answer
1






active

oldest

votes


















1












$begingroup$

If $v=(3,2,1)$ and $w=(5,4,6)$, then, indeed $vtimes w=(8,-13,2)$. And$$A.(vtimes w)=left(4+frac{13sqrt{3}}2,4sqrt{3}-frac{13}2,2right).$$It turns out thatbegin{align}(Av)times(Aw)&=left(frac{3}{2}-sqrt{3},1+frac{3 sqrt{3}}{2},1right)timesleft(frac{5}{2}-2 sqrt{3},2+frac{5 sqrt{3}}{2},6right)\&=left(4+frac{13sqrt{3}}2,4sqrt{3}-frac{13}2,2right).end{align}As you can see, $(Av)times(Aw)=det(A).A(vtimes w)$ in this case. Actually, it's true for every orthogonal matrix $A$.






share|cite|improve this answer









$endgroup$













  • $begingroup$
    Any hints on how to prove this fact that $Av times Aw=det(A)A(vtimes w)$ when $A$ is orthogonal?
    $endgroup$
    – Al Jebr
    Jan 28 at 19:58












  • $begingroup$
    You will find a proof here.
    $endgroup$
    – José Carlos Santos
    Jan 28 at 21:10












Your Answer





StackExchange.ifUsing("editor", function () {
return StackExchange.using("mathjaxEditing", function () {
StackExchange.MarkdownEditor.creationCallbacks.add(function (editor, postfix) {
StackExchange.mathjaxEditing.prepareWmdForMathJax(editor, postfix, [["$", "$"], ["\\(","\\)"]]);
});
});
}, "mathjax-editing");

StackExchange.ready(function() {
var channelOptions = {
tags: "".split(" "),
id: "69"
};
initTagRenderer("".split(" "), "".split(" "), channelOptions);

StackExchange.using("externalEditor", function() {
// Have to fire editor after snippets, if snippets enabled
if (StackExchange.settings.snippets.snippetsEnabled) {
StackExchange.using("snippets", function() {
createEditor();
});
}
else {
createEditor();
}
});

function createEditor() {
StackExchange.prepareEditor({
heartbeatType: 'answer',
autoActivateHeartbeat: false,
convertImagesToLinks: true,
noModals: true,
showLowRepImageUploadWarning: true,
reputationToPostImages: 10,
bindNavPrevention: true,
postfix: "",
imageUploader: {
brandingHtml: "Powered by u003ca class="icon-imgur-white" href="https://imgur.com/"u003eu003c/au003e",
contentPolicyHtml: "User contributions licensed under u003ca href="https://creativecommons.org/licenses/by-sa/3.0/"u003ecc by-sa 3.0 with attribution requiredu003c/au003e u003ca href="https://stackoverflow.com/legal/content-policy"u003e(content policy)u003c/au003e",
allowUrls: true
},
noCode: true, onDemand: true,
discardSelector: ".discard-answer"
,immediatelyShowMarkdownHelp:true
});


}
});














draft saved

draft discarded


















StackExchange.ready(
function () {
StackExchange.openid.initPostLogin('.new-post-login', 'https%3a%2f%2fmath.stackexchange.com%2fquestions%2f3091306%2fwhat-does-it-mean-for-the-cross-product-to-be-invariant-under-orthogonal-transfo%23new-answer', 'question_page');
}
);

Post as a guest















Required, but never shown

























1 Answer
1






active

oldest

votes








1 Answer
1






active

oldest

votes









active

oldest

votes






active

oldest

votes









1












$begingroup$

If $v=(3,2,1)$ and $w=(5,4,6)$, then, indeed $vtimes w=(8,-13,2)$. And$$A.(vtimes w)=left(4+frac{13sqrt{3}}2,4sqrt{3}-frac{13}2,2right).$$It turns out thatbegin{align}(Av)times(Aw)&=left(frac{3}{2}-sqrt{3},1+frac{3 sqrt{3}}{2},1right)timesleft(frac{5}{2}-2 sqrt{3},2+frac{5 sqrt{3}}{2},6right)\&=left(4+frac{13sqrt{3}}2,4sqrt{3}-frac{13}2,2right).end{align}As you can see, $(Av)times(Aw)=det(A).A(vtimes w)$ in this case. Actually, it's true for every orthogonal matrix $A$.






share|cite|improve this answer









$endgroup$













  • $begingroup$
    Any hints on how to prove this fact that $Av times Aw=det(A)A(vtimes w)$ when $A$ is orthogonal?
    $endgroup$
    – Al Jebr
    Jan 28 at 19:58












  • $begingroup$
    You will find a proof here.
    $endgroup$
    – José Carlos Santos
    Jan 28 at 21:10
















1












$begingroup$

If $v=(3,2,1)$ and $w=(5,4,6)$, then, indeed $vtimes w=(8,-13,2)$. And$$A.(vtimes w)=left(4+frac{13sqrt{3}}2,4sqrt{3}-frac{13}2,2right).$$It turns out thatbegin{align}(Av)times(Aw)&=left(frac{3}{2}-sqrt{3},1+frac{3 sqrt{3}}{2},1right)timesleft(frac{5}{2}-2 sqrt{3},2+frac{5 sqrt{3}}{2},6right)\&=left(4+frac{13sqrt{3}}2,4sqrt{3}-frac{13}2,2right).end{align}As you can see, $(Av)times(Aw)=det(A).A(vtimes w)$ in this case. Actually, it's true for every orthogonal matrix $A$.






share|cite|improve this answer









$endgroup$













  • $begingroup$
    Any hints on how to prove this fact that $Av times Aw=det(A)A(vtimes w)$ when $A$ is orthogonal?
    $endgroup$
    – Al Jebr
    Jan 28 at 19:58












  • $begingroup$
    You will find a proof here.
    $endgroup$
    – José Carlos Santos
    Jan 28 at 21:10














1












1








1





$begingroup$

If $v=(3,2,1)$ and $w=(5,4,6)$, then, indeed $vtimes w=(8,-13,2)$. And$$A.(vtimes w)=left(4+frac{13sqrt{3}}2,4sqrt{3}-frac{13}2,2right).$$It turns out thatbegin{align}(Av)times(Aw)&=left(frac{3}{2}-sqrt{3},1+frac{3 sqrt{3}}{2},1right)timesleft(frac{5}{2}-2 sqrt{3},2+frac{5 sqrt{3}}{2},6right)\&=left(4+frac{13sqrt{3}}2,4sqrt{3}-frac{13}2,2right).end{align}As you can see, $(Av)times(Aw)=det(A).A(vtimes w)$ in this case. Actually, it's true for every orthogonal matrix $A$.






share|cite|improve this answer









$endgroup$



If $v=(3,2,1)$ and $w=(5,4,6)$, then, indeed $vtimes w=(8,-13,2)$. And$$A.(vtimes w)=left(4+frac{13sqrt{3}}2,4sqrt{3}-frac{13}2,2right).$$It turns out thatbegin{align}(Av)times(Aw)&=left(frac{3}{2}-sqrt{3},1+frac{3 sqrt{3}}{2},1right)timesleft(frac{5}{2}-2 sqrt{3},2+frac{5 sqrt{3}}{2},6right)\&=left(4+frac{13sqrt{3}}2,4sqrt{3}-frac{13}2,2right).end{align}As you can see, $(Av)times(Aw)=det(A).A(vtimes w)$ in this case. Actually, it's true for every orthogonal matrix $A$.







share|cite|improve this answer












share|cite|improve this answer



share|cite|improve this answer










answered Jan 28 at 19:55









José Carlos SantosJosé Carlos Santos

171k23132240




171k23132240












  • $begingroup$
    Any hints on how to prove this fact that $Av times Aw=det(A)A(vtimes w)$ when $A$ is orthogonal?
    $endgroup$
    – Al Jebr
    Jan 28 at 19:58












  • $begingroup$
    You will find a proof here.
    $endgroup$
    – José Carlos Santos
    Jan 28 at 21:10


















  • $begingroup$
    Any hints on how to prove this fact that $Av times Aw=det(A)A(vtimes w)$ when $A$ is orthogonal?
    $endgroup$
    – Al Jebr
    Jan 28 at 19:58












  • $begingroup$
    You will find a proof here.
    $endgroup$
    – José Carlos Santos
    Jan 28 at 21:10
















$begingroup$
Any hints on how to prove this fact that $Av times Aw=det(A)A(vtimes w)$ when $A$ is orthogonal?
$endgroup$
– Al Jebr
Jan 28 at 19:58






$begingroup$
Any hints on how to prove this fact that $Av times Aw=det(A)A(vtimes w)$ when $A$ is orthogonal?
$endgroup$
– Al Jebr
Jan 28 at 19:58














$begingroup$
You will find a proof here.
$endgroup$
– José Carlos Santos
Jan 28 at 21:10




$begingroup$
You will find a proof here.
$endgroup$
– José Carlos Santos
Jan 28 at 21:10


















draft saved

draft discarded




















































Thanks for contributing an answer to Mathematics Stack Exchange!


  • Please be sure to answer the question. Provide details and share your research!

But avoid



  • Asking for help, clarification, or responding to other answers.

  • Making statements based on opinion; back them up with references or personal experience.


Use MathJax to format equations. MathJax reference.


To learn more, see our tips on writing great answers.




draft saved


draft discarded














StackExchange.ready(
function () {
StackExchange.openid.initPostLogin('.new-post-login', 'https%3a%2f%2fmath.stackexchange.com%2fquestions%2f3091306%2fwhat-does-it-mean-for-the-cross-product-to-be-invariant-under-orthogonal-transfo%23new-answer', 'question_page');
}
);

Post as a guest















Required, but never shown





















































Required, but never shown














Required, but never shown












Required, but never shown







Required, but never shown

































Required, but never shown














Required, but never shown












Required, but never shown







Required, but never shown







Popular posts from this blog

Can a sorcerer learn a 5th-level spell early by creating spell slots using the Font of Magic feature?

ts Property 'filter' does not exist on type '{}'

Notepad++ export/extract a list of installed plugins